If $ageq 2$, $anmid b$, and $a^n-1mid b^n-1$ for all $ninmathbb{N}$, then $b=1$












7














Let $a,binmathbb{N}$ be such that $ageq 2$, $anmid b$, and $a^n-1mid b^n-1$ for all $ninmathbb{N}$, then $b=1$.



PS: In fact, if we do not assume that $anmid b$, then the statement should be $b=a^k$ for some $kinmathbb{N}cup{0}$. The above statement can deduce the fact.










share|cite|improve this question
























  • Please, try to make the titles of your questions more informative. E.g., Why does $a<b$ imply $a+c<b+c$? is much more useful for other users than A question about inequality. From How can I ask a good question?: Make your title as descriptive as possible. In many cases one can actually phrase the title as the question, at least in such a way so as to be comprehensible to an expert reader. You can find more tips for choosing a good title here.
    – Martin Sleziak
    Oct 12 '16 at 5:23
















7














Let $a,binmathbb{N}$ be such that $ageq 2$, $anmid b$, and $a^n-1mid b^n-1$ for all $ninmathbb{N}$, then $b=1$.



PS: In fact, if we do not assume that $anmid b$, then the statement should be $b=a^k$ for some $kinmathbb{N}cup{0}$. The above statement can deduce the fact.










share|cite|improve this question
























  • Please, try to make the titles of your questions more informative. E.g., Why does $a<b$ imply $a+c<b+c$? is much more useful for other users than A question about inequality. From How can I ask a good question?: Make your title as descriptive as possible. In many cases one can actually phrase the title as the question, at least in such a way so as to be comprehensible to an expert reader. You can find more tips for choosing a good title here.
    – Martin Sleziak
    Oct 12 '16 at 5:23














7












7








7


1





Let $a,binmathbb{N}$ be such that $ageq 2$, $anmid b$, and $a^n-1mid b^n-1$ for all $ninmathbb{N}$, then $b=1$.



PS: In fact, if we do not assume that $anmid b$, then the statement should be $b=a^k$ for some $kinmathbb{N}cup{0}$. The above statement can deduce the fact.










share|cite|improve this question















Let $a,binmathbb{N}$ be such that $ageq 2$, $anmid b$, and $a^n-1mid b^n-1$ for all $ninmathbb{N}$, then $b=1$.



PS: In fact, if we do not assume that $anmid b$, then the statement should be $b=a^k$ for some $kinmathbb{N}cup{0}$. The above statement can deduce the fact.







elementary-number-theory divisibility






share|cite|improve this question















share|cite|improve this question













share|cite|improve this question




share|cite|improve this question








edited Oct 12 '16 at 5:23









Martin Sleziak

44.7k8115271




44.7k8115271










asked Oct 12 '16 at 4:14









m-agag2016

63448




63448












  • Please, try to make the titles of your questions more informative. E.g., Why does $a<b$ imply $a+c<b+c$? is much more useful for other users than A question about inequality. From How can I ask a good question?: Make your title as descriptive as possible. In many cases one can actually phrase the title as the question, at least in such a way so as to be comprehensible to an expert reader. You can find more tips for choosing a good title here.
    – Martin Sleziak
    Oct 12 '16 at 5:23


















  • Please, try to make the titles of your questions more informative. E.g., Why does $a<b$ imply $a+c<b+c$? is much more useful for other users than A question about inequality. From How can I ask a good question?: Make your title as descriptive as possible. In many cases one can actually phrase the title as the question, at least in such a way so as to be comprehensible to an expert reader. You can find more tips for choosing a good title here.
    – Martin Sleziak
    Oct 12 '16 at 5:23
















Please, try to make the titles of your questions more informative. E.g., Why does $a<b$ imply $a+c<b+c$? is much more useful for other users than A question about inequality. From How can I ask a good question?: Make your title as descriptive as possible. In many cases one can actually phrase the title as the question, at least in such a way so as to be comprehensible to an expert reader. You can find more tips for choosing a good title here.
– Martin Sleziak
Oct 12 '16 at 5:23




Please, try to make the titles of your questions more informative. E.g., Why does $a<b$ imply $a+c<b+c$? is much more useful for other users than A question about inequality. From How can I ask a good question?: Make your title as descriptive as possible. In many cases one can actually phrase the title as the question, at least in such a way so as to be comprehensible to an expert reader. You can find more tips for choosing a good title here.
– Martin Sleziak
Oct 12 '16 at 5:23










3 Answers
3






active

oldest

votes


















2














This is a solution here which was found in some paper in AMM. it's a very nice proof. but it's very constructive. For example, the construction of $Q_k(x)$, $p_k$ and $r_{k,n}$, it's not so easy to construct them. Maybe there is some direct method.



This is a solution here which was found in some paper in AMM. it's a very nice proof. but it's very constructive. For example, the construction of $Q_k(x)$, $p_k$ and $r_{k,n}$, it's not so easy to construct them. Maybe there is some direct method.






share|cite|improve this answer





























    0














    Here is my proof from AoPS.



    $textbf{Problem 1.}$ Prove that if $b^n-1mid a^n-1$ for all $n$, then $a=b^k$ for some integer $k$.



    $textbf{Lemma 1.}$ If $b^n-1mid a^n-1$ for all $n$, then $(b-1)(b^2-1)ldots (b^n-1)mid n!(a-1)(a-b)ldots (a-b^{n-1})$ for every $n$.



    $textbf{Proof of Lemma 1.}$ First consider any prime $pmid (b-1)(b^2-1)ldots (b^n-1)$, so there exists minimal $d$ such that $pmid b^d-1$ and it's well-known fact that then $x^d-1equiv (x-1)(x-b)ldots (x-b^{d-1})pmod{p^{nu_p(b^d-1)}}$, so $P(x)=x^d-1$ has exactly $d$ consecutive roots $1, b, b^2,ldots , b^{d-1}$. From the condition $b^d-1mid a^d-1$ we get that $a$ is also the root of polynomial $x^d-1$. So easy to see that there exists only one $0leq d'leq d$, such that $a-b^{d'}equiv 0pmod{p^{nu_p(b^d-1)}}$. From this we get that for all $iequiv d'pmod{d}$, $a-b^iequiv 0pmod{p^{nu_p(b^d-1)}}$. Now from LTE lemma we get that $nu_p(b^{jd}-1)leqnu_p(b^d-1)+nu_p(j)$, for each $j$, so $nu_p(prod_{i=1}^n(b^i-1))leq[n/d]nu_p(b^d-1)+nu_p(n!)leqnu_p(n!(a-1)(a-b)ldots (a-b^{n-1}))$. Combining similar inequalities for every $p$ we get Lemma 1. $Box$



    $textbf{Proof of Problem 1.}$ Firstly it's simple exercise to prove that if a prime $p$ is such that $pmid a$, then $pmid b$. So there exists a positive integer $t$, such that $anmid b^{t-1}$, but $amid b^t$. From Lemma 1 we get that $(2t)!frac{(a-1)(a-b)ldots (a-b^{t-1})(a-b^t)ldots (a-b^{2t-1})}{(b-1)(b^2-1)ldots (b^t-1)(b^{t+1}-1)ldots (b^{2t}-1)}inmathbb{Z}$. But for every $i$ we have that $gcd(a, b^i-1)$, so if we just look on the numerator of the fraction above then we get that $$gcd(a,1)gcd(a, b)ldots gcd(a, b^{t-1})a^tmid (2t)!frac{(a-1)(a-b)ldots (a-b^{t-1})(a-b^t)ldots (a-b^{2t-1})}{(b-1)(b^2-1)ldots (b^t-1)(b^{t+1}-1)ldots (b^{2t}-1)}$$ and if $anot= b^x$ for every natural $x$, then $$|(2t)!frac{(a-1)(a-b)ldots (a-b^{t-1})(a-b^t)ldots (a-b^{2t-1})}{(b-1)(b^2-1)ldots (b^t-1)(b^{t+1}-1)ldots (b^{2t}-1)}|geq gcd(a,1)gcd(a, b)ldots gcd(a, b^{t-1})a^t$$ But it's easy to check that $|(2t)!frac{(a-1)(a-b)ldots (a-b^{t-1})(a-b^t)ldots (a-b^{2t-1})}{(b-1)(b^2-1)ldots (b^t-1)(b^{t+1}-1)ldots (b^{2t}-1)}|leq (2t)!a^t$ and $gcd(a,1)gcd(a, b)ldots gcd(a, b^{t-1})geq 2^{frac{t(t-1)}{2}}$ (here we used the fact that $gcd(a, b^i)midgcd(a, b^{i+1})$ and $gcd(a, b^{i-1})not=gcd(a, b^i)$ for $ileq t-1$). So $2^{frac{t(t-1)}{2}}a^tleq (2t)!a^t$. Contradiction. $Box$






    share|cite|improve this answer























    • Can someone find any mistakes in another proof : Let $p$ be a prime number greater than $a$. Consider a polynomial $F(x)=x^p-1$ over $mod{frac{b^p-1}{b-1}}$. One can get that it has $d$ consecutive roots $1, b,ldots , b^{p-1}$. So from the fact $b^p-1mid a^p-1$ we get that $a$ is also root of $F(x)equiv 0pmod{frac{b^p-1}{b-1}}$. Thus, $aequiv b^ipmod{frac{b^p-1}{b-1}}$ for some $ileq p-1$.
      – solver6
      Nov 27 '18 at 21:19










    • Can someone find any mistakes in another proof : Let $p$ be a prime number greater than $a$. Consider a polynomial $F(x)=x^p-1$ over $mod{frac{b^p-1}{b-1}}$. One can get that $P(x)$ has $d$ consecutive roots $1, b,ldots , b^{p-1}$. So from the fact $b^p-1mid a^p-1$ we get that $a$ is also root of $F(x)equiv 0pmod{frac{b^p-1}{b-1}}$. Thus, $aequiv b^ipmod{frac{b^p-1}{b-1}}$ for some $ileq p-1$. Finally, if $anot= b^i$, then $b^{p-1}-1geq |a-b^i|geqfrac{b^p-1}{b-1}$. Contradiction. $Box$
      – solver6
      Nov 27 '18 at 21:26





















    0














    Reference for a generalization of this theorem and it's proof in French. See this article.



    Langlois, Bruno, An arithmetical application of Hadamard’s quotient theorem, Ann. Univ. Sci. Budap. Rolando Eötvös, Sect. Comput. 44, 183-196 (2015). ZBL1389.11046.






    share|cite|improve this answer





















      Your Answer





      StackExchange.ifUsing("editor", function () {
      return StackExchange.using("mathjaxEditing", function () {
      StackExchange.MarkdownEditor.creationCallbacks.add(function (editor, postfix) {
      StackExchange.mathjaxEditing.prepareWmdForMathJax(editor, postfix, [["$", "$"], ["\\(","\\)"]]);
      });
      });
      }, "mathjax-editing");

      StackExchange.ready(function() {
      var channelOptions = {
      tags: "".split(" "),
      id: "69"
      };
      initTagRenderer("".split(" "), "".split(" "), channelOptions);

      StackExchange.using("externalEditor", function() {
      // Have to fire editor after snippets, if snippets enabled
      if (StackExchange.settings.snippets.snippetsEnabled) {
      StackExchange.using("snippets", function() {
      createEditor();
      });
      }
      else {
      createEditor();
      }
      });

      function createEditor() {
      StackExchange.prepareEditor({
      heartbeatType: 'answer',
      autoActivateHeartbeat: false,
      convertImagesToLinks: true,
      noModals: true,
      showLowRepImageUploadWarning: true,
      reputationToPostImages: 10,
      bindNavPrevention: true,
      postfix: "",
      imageUploader: {
      brandingHtml: "Powered by u003ca class="icon-imgur-white" href="https://imgur.com/"u003eu003c/au003e",
      contentPolicyHtml: "User contributions licensed under u003ca href="https://creativecommons.org/licenses/by-sa/3.0/"u003ecc by-sa 3.0 with attribution requiredu003c/au003e u003ca href="https://stackoverflow.com/legal/content-policy"u003e(content policy)u003c/au003e",
      allowUrls: true
      },
      noCode: true, onDemand: true,
      discardSelector: ".discard-answer"
      ,immediatelyShowMarkdownHelp:true
      });


      }
      });














      draft saved

      draft discarded


















      StackExchange.ready(
      function () {
      StackExchange.openid.initPostLogin('.new-post-login', 'https%3a%2f%2fmath.stackexchange.com%2fquestions%2f1964788%2fif-a-geq-2-a-nmid-b-and-an-1-mid-bn-1-for-all-n-in-mathbbn-then%23new-answer', 'question_page');
      }
      );

      Post as a guest















      Required, but never shown

























      3 Answers
      3






      active

      oldest

      votes








      3 Answers
      3






      active

      oldest

      votes









      active

      oldest

      votes






      active

      oldest

      votes









      2














      This is a solution here which was found in some paper in AMM. it's a very nice proof. but it's very constructive. For example, the construction of $Q_k(x)$, $p_k$ and $r_{k,n}$, it's not so easy to construct them. Maybe there is some direct method.



      This is a solution here which was found in some paper in AMM. it's a very nice proof. but it's very constructive. For example, the construction of $Q_k(x)$, $p_k$ and $r_{k,n}$, it's not so easy to construct them. Maybe there is some direct method.






      share|cite|improve this answer


























        2














        This is a solution here which was found in some paper in AMM. it's a very nice proof. but it's very constructive. For example, the construction of $Q_k(x)$, $p_k$ and $r_{k,n}$, it's not so easy to construct them. Maybe there is some direct method.



        This is a solution here which was found in some paper in AMM. it's a very nice proof. but it's very constructive. For example, the construction of $Q_k(x)$, $p_k$ and $r_{k,n}$, it's not so easy to construct them. Maybe there is some direct method.






        share|cite|improve this answer
























          2












          2








          2






          This is a solution here which was found in some paper in AMM. it's a very nice proof. but it's very constructive. For example, the construction of $Q_k(x)$, $p_k$ and $r_{k,n}$, it's not so easy to construct them. Maybe there is some direct method.



          This is a solution here which was found in some paper in AMM. it's a very nice proof. but it's very constructive. For example, the construction of $Q_k(x)$, $p_k$ and $r_{k,n}$, it's not so easy to construct them. Maybe there is some direct method.






          share|cite|improve this answer












          This is a solution here which was found in some paper in AMM. it's a very nice proof. but it's very constructive. For example, the construction of $Q_k(x)$, $p_k$ and $r_{k,n}$, it's not so easy to construct them. Maybe there is some direct method.



          This is a solution here which was found in some paper in AMM. it's a very nice proof. but it's very constructive. For example, the construction of $Q_k(x)$, $p_k$ and $r_{k,n}$, it's not so easy to construct them. Maybe there is some direct method.







          share|cite|improve this answer












          share|cite|improve this answer



          share|cite|improve this answer










          answered Nov 20 '16 at 4:27









          m-agag2016

          63448




          63448























              0














              Here is my proof from AoPS.



              $textbf{Problem 1.}$ Prove that if $b^n-1mid a^n-1$ for all $n$, then $a=b^k$ for some integer $k$.



              $textbf{Lemma 1.}$ If $b^n-1mid a^n-1$ for all $n$, then $(b-1)(b^2-1)ldots (b^n-1)mid n!(a-1)(a-b)ldots (a-b^{n-1})$ for every $n$.



              $textbf{Proof of Lemma 1.}$ First consider any prime $pmid (b-1)(b^2-1)ldots (b^n-1)$, so there exists minimal $d$ such that $pmid b^d-1$ and it's well-known fact that then $x^d-1equiv (x-1)(x-b)ldots (x-b^{d-1})pmod{p^{nu_p(b^d-1)}}$, so $P(x)=x^d-1$ has exactly $d$ consecutive roots $1, b, b^2,ldots , b^{d-1}$. From the condition $b^d-1mid a^d-1$ we get that $a$ is also the root of polynomial $x^d-1$. So easy to see that there exists only one $0leq d'leq d$, such that $a-b^{d'}equiv 0pmod{p^{nu_p(b^d-1)}}$. From this we get that for all $iequiv d'pmod{d}$, $a-b^iequiv 0pmod{p^{nu_p(b^d-1)}}$. Now from LTE lemma we get that $nu_p(b^{jd}-1)leqnu_p(b^d-1)+nu_p(j)$, for each $j$, so $nu_p(prod_{i=1}^n(b^i-1))leq[n/d]nu_p(b^d-1)+nu_p(n!)leqnu_p(n!(a-1)(a-b)ldots (a-b^{n-1}))$. Combining similar inequalities for every $p$ we get Lemma 1. $Box$



              $textbf{Proof of Problem 1.}$ Firstly it's simple exercise to prove that if a prime $p$ is such that $pmid a$, then $pmid b$. So there exists a positive integer $t$, such that $anmid b^{t-1}$, but $amid b^t$. From Lemma 1 we get that $(2t)!frac{(a-1)(a-b)ldots (a-b^{t-1})(a-b^t)ldots (a-b^{2t-1})}{(b-1)(b^2-1)ldots (b^t-1)(b^{t+1}-1)ldots (b^{2t}-1)}inmathbb{Z}$. But for every $i$ we have that $gcd(a, b^i-1)$, so if we just look on the numerator of the fraction above then we get that $$gcd(a,1)gcd(a, b)ldots gcd(a, b^{t-1})a^tmid (2t)!frac{(a-1)(a-b)ldots (a-b^{t-1})(a-b^t)ldots (a-b^{2t-1})}{(b-1)(b^2-1)ldots (b^t-1)(b^{t+1}-1)ldots (b^{2t}-1)}$$ and if $anot= b^x$ for every natural $x$, then $$|(2t)!frac{(a-1)(a-b)ldots (a-b^{t-1})(a-b^t)ldots (a-b^{2t-1})}{(b-1)(b^2-1)ldots (b^t-1)(b^{t+1}-1)ldots (b^{2t}-1)}|geq gcd(a,1)gcd(a, b)ldots gcd(a, b^{t-1})a^t$$ But it's easy to check that $|(2t)!frac{(a-1)(a-b)ldots (a-b^{t-1})(a-b^t)ldots (a-b^{2t-1})}{(b-1)(b^2-1)ldots (b^t-1)(b^{t+1}-1)ldots (b^{2t}-1)}|leq (2t)!a^t$ and $gcd(a,1)gcd(a, b)ldots gcd(a, b^{t-1})geq 2^{frac{t(t-1)}{2}}$ (here we used the fact that $gcd(a, b^i)midgcd(a, b^{i+1})$ and $gcd(a, b^{i-1})not=gcd(a, b^i)$ for $ileq t-1$). So $2^{frac{t(t-1)}{2}}a^tleq (2t)!a^t$. Contradiction. $Box$






              share|cite|improve this answer























              • Can someone find any mistakes in another proof : Let $p$ be a prime number greater than $a$. Consider a polynomial $F(x)=x^p-1$ over $mod{frac{b^p-1}{b-1}}$. One can get that it has $d$ consecutive roots $1, b,ldots , b^{p-1}$. So from the fact $b^p-1mid a^p-1$ we get that $a$ is also root of $F(x)equiv 0pmod{frac{b^p-1}{b-1}}$. Thus, $aequiv b^ipmod{frac{b^p-1}{b-1}}$ for some $ileq p-1$.
                – solver6
                Nov 27 '18 at 21:19










              • Can someone find any mistakes in another proof : Let $p$ be a prime number greater than $a$. Consider a polynomial $F(x)=x^p-1$ over $mod{frac{b^p-1}{b-1}}$. One can get that $P(x)$ has $d$ consecutive roots $1, b,ldots , b^{p-1}$. So from the fact $b^p-1mid a^p-1$ we get that $a$ is also root of $F(x)equiv 0pmod{frac{b^p-1}{b-1}}$. Thus, $aequiv b^ipmod{frac{b^p-1}{b-1}}$ for some $ileq p-1$. Finally, if $anot= b^i$, then $b^{p-1}-1geq |a-b^i|geqfrac{b^p-1}{b-1}$. Contradiction. $Box$
                – solver6
                Nov 27 '18 at 21:26


















              0














              Here is my proof from AoPS.



              $textbf{Problem 1.}$ Prove that if $b^n-1mid a^n-1$ for all $n$, then $a=b^k$ for some integer $k$.



              $textbf{Lemma 1.}$ If $b^n-1mid a^n-1$ for all $n$, then $(b-1)(b^2-1)ldots (b^n-1)mid n!(a-1)(a-b)ldots (a-b^{n-1})$ for every $n$.



              $textbf{Proof of Lemma 1.}$ First consider any prime $pmid (b-1)(b^2-1)ldots (b^n-1)$, so there exists minimal $d$ such that $pmid b^d-1$ and it's well-known fact that then $x^d-1equiv (x-1)(x-b)ldots (x-b^{d-1})pmod{p^{nu_p(b^d-1)}}$, so $P(x)=x^d-1$ has exactly $d$ consecutive roots $1, b, b^2,ldots , b^{d-1}$. From the condition $b^d-1mid a^d-1$ we get that $a$ is also the root of polynomial $x^d-1$. So easy to see that there exists only one $0leq d'leq d$, such that $a-b^{d'}equiv 0pmod{p^{nu_p(b^d-1)}}$. From this we get that for all $iequiv d'pmod{d}$, $a-b^iequiv 0pmod{p^{nu_p(b^d-1)}}$. Now from LTE lemma we get that $nu_p(b^{jd}-1)leqnu_p(b^d-1)+nu_p(j)$, for each $j$, so $nu_p(prod_{i=1}^n(b^i-1))leq[n/d]nu_p(b^d-1)+nu_p(n!)leqnu_p(n!(a-1)(a-b)ldots (a-b^{n-1}))$. Combining similar inequalities for every $p$ we get Lemma 1. $Box$



              $textbf{Proof of Problem 1.}$ Firstly it's simple exercise to prove that if a prime $p$ is such that $pmid a$, then $pmid b$. So there exists a positive integer $t$, such that $anmid b^{t-1}$, but $amid b^t$. From Lemma 1 we get that $(2t)!frac{(a-1)(a-b)ldots (a-b^{t-1})(a-b^t)ldots (a-b^{2t-1})}{(b-1)(b^2-1)ldots (b^t-1)(b^{t+1}-1)ldots (b^{2t}-1)}inmathbb{Z}$. But for every $i$ we have that $gcd(a, b^i-1)$, so if we just look on the numerator of the fraction above then we get that $$gcd(a,1)gcd(a, b)ldots gcd(a, b^{t-1})a^tmid (2t)!frac{(a-1)(a-b)ldots (a-b^{t-1})(a-b^t)ldots (a-b^{2t-1})}{(b-1)(b^2-1)ldots (b^t-1)(b^{t+1}-1)ldots (b^{2t}-1)}$$ and if $anot= b^x$ for every natural $x$, then $$|(2t)!frac{(a-1)(a-b)ldots (a-b^{t-1})(a-b^t)ldots (a-b^{2t-1})}{(b-1)(b^2-1)ldots (b^t-1)(b^{t+1}-1)ldots (b^{2t}-1)}|geq gcd(a,1)gcd(a, b)ldots gcd(a, b^{t-1})a^t$$ But it's easy to check that $|(2t)!frac{(a-1)(a-b)ldots (a-b^{t-1})(a-b^t)ldots (a-b^{2t-1})}{(b-1)(b^2-1)ldots (b^t-1)(b^{t+1}-1)ldots (b^{2t}-1)}|leq (2t)!a^t$ and $gcd(a,1)gcd(a, b)ldots gcd(a, b^{t-1})geq 2^{frac{t(t-1)}{2}}$ (here we used the fact that $gcd(a, b^i)midgcd(a, b^{i+1})$ and $gcd(a, b^{i-1})not=gcd(a, b^i)$ for $ileq t-1$). So $2^{frac{t(t-1)}{2}}a^tleq (2t)!a^t$. Contradiction. $Box$






              share|cite|improve this answer























              • Can someone find any mistakes in another proof : Let $p$ be a prime number greater than $a$. Consider a polynomial $F(x)=x^p-1$ over $mod{frac{b^p-1}{b-1}}$. One can get that it has $d$ consecutive roots $1, b,ldots , b^{p-1}$. So from the fact $b^p-1mid a^p-1$ we get that $a$ is also root of $F(x)equiv 0pmod{frac{b^p-1}{b-1}}$. Thus, $aequiv b^ipmod{frac{b^p-1}{b-1}}$ for some $ileq p-1$.
                – solver6
                Nov 27 '18 at 21:19










              • Can someone find any mistakes in another proof : Let $p$ be a prime number greater than $a$. Consider a polynomial $F(x)=x^p-1$ over $mod{frac{b^p-1}{b-1}}$. One can get that $P(x)$ has $d$ consecutive roots $1, b,ldots , b^{p-1}$. So from the fact $b^p-1mid a^p-1$ we get that $a$ is also root of $F(x)equiv 0pmod{frac{b^p-1}{b-1}}$. Thus, $aequiv b^ipmod{frac{b^p-1}{b-1}}$ for some $ileq p-1$. Finally, if $anot= b^i$, then $b^{p-1}-1geq |a-b^i|geqfrac{b^p-1}{b-1}$. Contradiction. $Box$
                – solver6
                Nov 27 '18 at 21:26
















              0












              0








              0






              Here is my proof from AoPS.



              $textbf{Problem 1.}$ Prove that if $b^n-1mid a^n-1$ for all $n$, then $a=b^k$ for some integer $k$.



              $textbf{Lemma 1.}$ If $b^n-1mid a^n-1$ for all $n$, then $(b-1)(b^2-1)ldots (b^n-1)mid n!(a-1)(a-b)ldots (a-b^{n-1})$ for every $n$.



              $textbf{Proof of Lemma 1.}$ First consider any prime $pmid (b-1)(b^2-1)ldots (b^n-1)$, so there exists minimal $d$ such that $pmid b^d-1$ and it's well-known fact that then $x^d-1equiv (x-1)(x-b)ldots (x-b^{d-1})pmod{p^{nu_p(b^d-1)}}$, so $P(x)=x^d-1$ has exactly $d$ consecutive roots $1, b, b^2,ldots , b^{d-1}$. From the condition $b^d-1mid a^d-1$ we get that $a$ is also the root of polynomial $x^d-1$. So easy to see that there exists only one $0leq d'leq d$, such that $a-b^{d'}equiv 0pmod{p^{nu_p(b^d-1)}}$. From this we get that for all $iequiv d'pmod{d}$, $a-b^iequiv 0pmod{p^{nu_p(b^d-1)}}$. Now from LTE lemma we get that $nu_p(b^{jd}-1)leqnu_p(b^d-1)+nu_p(j)$, for each $j$, so $nu_p(prod_{i=1}^n(b^i-1))leq[n/d]nu_p(b^d-1)+nu_p(n!)leqnu_p(n!(a-1)(a-b)ldots (a-b^{n-1}))$. Combining similar inequalities for every $p$ we get Lemma 1. $Box$



              $textbf{Proof of Problem 1.}$ Firstly it's simple exercise to prove that if a prime $p$ is such that $pmid a$, then $pmid b$. So there exists a positive integer $t$, such that $anmid b^{t-1}$, but $amid b^t$. From Lemma 1 we get that $(2t)!frac{(a-1)(a-b)ldots (a-b^{t-1})(a-b^t)ldots (a-b^{2t-1})}{(b-1)(b^2-1)ldots (b^t-1)(b^{t+1}-1)ldots (b^{2t}-1)}inmathbb{Z}$. But for every $i$ we have that $gcd(a, b^i-1)$, so if we just look on the numerator of the fraction above then we get that $$gcd(a,1)gcd(a, b)ldots gcd(a, b^{t-1})a^tmid (2t)!frac{(a-1)(a-b)ldots (a-b^{t-1})(a-b^t)ldots (a-b^{2t-1})}{(b-1)(b^2-1)ldots (b^t-1)(b^{t+1}-1)ldots (b^{2t}-1)}$$ and if $anot= b^x$ for every natural $x$, then $$|(2t)!frac{(a-1)(a-b)ldots (a-b^{t-1})(a-b^t)ldots (a-b^{2t-1})}{(b-1)(b^2-1)ldots (b^t-1)(b^{t+1}-1)ldots (b^{2t}-1)}|geq gcd(a,1)gcd(a, b)ldots gcd(a, b^{t-1})a^t$$ But it's easy to check that $|(2t)!frac{(a-1)(a-b)ldots (a-b^{t-1})(a-b^t)ldots (a-b^{2t-1})}{(b-1)(b^2-1)ldots (b^t-1)(b^{t+1}-1)ldots (b^{2t}-1)}|leq (2t)!a^t$ and $gcd(a,1)gcd(a, b)ldots gcd(a, b^{t-1})geq 2^{frac{t(t-1)}{2}}$ (here we used the fact that $gcd(a, b^i)midgcd(a, b^{i+1})$ and $gcd(a, b^{i-1})not=gcd(a, b^i)$ for $ileq t-1$). So $2^{frac{t(t-1)}{2}}a^tleq (2t)!a^t$. Contradiction. $Box$






              share|cite|improve this answer














              Here is my proof from AoPS.



              $textbf{Problem 1.}$ Prove that if $b^n-1mid a^n-1$ for all $n$, then $a=b^k$ for some integer $k$.



              $textbf{Lemma 1.}$ If $b^n-1mid a^n-1$ for all $n$, then $(b-1)(b^2-1)ldots (b^n-1)mid n!(a-1)(a-b)ldots (a-b^{n-1})$ for every $n$.



              $textbf{Proof of Lemma 1.}$ First consider any prime $pmid (b-1)(b^2-1)ldots (b^n-1)$, so there exists minimal $d$ such that $pmid b^d-1$ and it's well-known fact that then $x^d-1equiv (x-1)(x-b)ldots (x-b^{d-1})pmod{p^{nu_p(b^d-1)}}$, so $P(x)=x^d-1$ has exactly $d$ consecutive roots $1, b, b^2,ldots , b^{d-1}$. From the condition $b^d-1mid a^d-1$ we get that $a$ is also the root of polynomial $x^d-1$. So easy to see that there exists only one $0leq d'leq d$, such that $a-b^{d'}equiv 0pmod{p^{nu_p(b^d-1)}}$. From this we get that for all $iequiv d'pmod{d}$, $a-b^iequiv 0pmod{p^{nu_p(b^d-1)}}$. Now from LTE lemma we get that $nu_p(b^{jd}-1)leqnu_p(b^d-1)+nu_p(j)$, for each $j$, so $nu_p(prod_{i=1}^n(b^i-1))leq[n/d]nu_p(b^d-1)+nu_p(n!)leqnu_p(n!(a-1)(a-b)ldots (a-b^{n-1}))$. Combining similar inequalities for every $p$ we get Lemma 1. $Box$



              $textbf{Proof of Problem 1.}$ Firstly it's simple exercise to prove that if a prime $p$ is such that $pmid a$, then $pmid b$. So there exists a positive integer $t$, such that $anmid b^{t-1}$, but $amid b^t$. From Lemma 1 we get that $(2t)!frac{(a-1)(a-b)ldots (a-b^{t-1})(a-b^t)ldots (a-b^{2t-1})}{(b-1)(b^2-1)ldots (b^t-1)(b^{t+1}-1)ldots (b^{2t}-1)}inmathbb{Z}$. But for every $i$ we have that $gcd(a, b^i-1)$, so if we just look on the numerator of the fraction above then we get that $$gcd(a,1)gcd(a, b)ldots gcd(a, b^{t-1})a^tmid (2t)!frac{(a-1)(a-b)ldots (a-b^{t-1})(a-b^t)ldots (a-b^{2t-1})}{(b-1)(b^2-1)ldots (b^t-1)(b^{t+1}-1)ldots (b^{2t}-1)}$$ and if $anot= b^x$ for every natural $x$, then $$|(2t)!frac{(a-1)(a-b)ldots (a-b^{t-1})(a-b^t)ldots (a-b^{2t-1})}{(b-1)(b^2-1)ldots (b^t-1)(b^{t+1}-1)ldots (b^{2t}-1)}|geq gcd(a,1)gcd(a, b)ldots gcd(a, b^{t-1})a^t$$ But it's easy to check that $|(2t)!frac{(a-1)(a-b)ldots (a-b^{t-1})(a-b^t)ldots (a-b^{2t-1})}{(b-1)(b^2-1)ldots (b^t-1)(b^{t+1}-1)ldots (b^{2t}-1)}|leq (2t)!a^t$ and $gcd(a,1)gcd(a, b)ldots gcd(a, b^{t-1})geq 2^{frac{t(t-1)}{2}}$ (here we used the fact that $gcd(a, b^i)midgcd(a, b^{i+1})$ and $gcd(a, b^{i-1})not=gcd(a, b^i)$ for $ileq t-1$). So $2^{frac{t(t-1)}{2}}a^tleq (2t)!a^t$. Contradiction. $Box$







              share|cite|improve this answer














              share|cite|improve this answer



              share|cite|improve this answer








              edited Nov 27 '18 at 20:38

























              answered Nov 27 '18 at 19:58









              solver6

              5917




              5917












              • Can someone find any mistakes in another proof : Let $p$ be a prime number greater than $a$. Consider a polynomial $F(x)=x^p-1$ over $mod{frac{b^p-1}{b-1}}$. One can get that it has $d$ consecutive roots $1, b,ldots , b^{p-1}$. So from the fact $b^p-1mid a^p-1$ we get that $a$ is also root of $F(x)equiv 0pmod{frac{b^p-1}{b-1}}$. Thus, $aequiv b^ipmod{frac{b^p-1}{b-1}}$ for some $ileq p-1$.
                – solver6
                Nov 27 '18 at 21:19










              • Can someone find any mistakes in another proof : Let $p$ be a prime number greater than $a$. Consider a polynomial $F(x)=x^p-1$ over $mod{frac{b^p-1}{b-1}}$. One can get that $P(x)$ has $d$ consecutive roots $1, b,ldots , b^{p-1}$. So from the fact $b^p-1mid a^p-1$ we get that $a$ is also root of $F(x)equiv 0pmod{frac{b^p-1}{b-1}}$. Thus, $aequiv b^ipmod{frac{b^p-1}{b-1}}$ for some $ileq p-1$. Finally, if $anot= b^i$, then $b^{p-1}-1geq |a-b^i|geqfrac{b^p-1}{b-1}$. Contradiction. $Box$
                – solver6
                Nov 27 '18 at 21:26




















              • Can someone find any mistakes in another proof : Let $p$ be a prime number greater than $a$. Consider a polynomial $F(x)=x^p-1$ over $mod{frac{b^p-1}{b-1}}$. One can get that it has $d$ consecutive roots $1, b,ldots , b^{p-1}$. So from the fact $b^p-1mid a^p-1$ we get that $a$ is also root of $F(x)equiv 0pmod{frac{b^p-1}{b-1}}$. Thus, $aequiv b^ipmod{frac{b^p-1}{b-1}}$ for some $ileq p-1$.
                – solver6
                Nov 27 '18 at 21:19










              • Can someone find any mistakes in another proof : Let $p$ be a prime number greater than $a$. Consider a polynomial $F(x)=x^p-1$ over $mod{frac{b^p-1}{b-1}}$. One can get that $P(x)$ has $d$ consecutive roots $1, b,ldots , b^{p-1}$. So from the fact $b^p-1mid a^p-1$ we get that $a$ is also root of $F(x)equiv 0pmod{frac{b^p-1}{b-1}}$. Thus, $aequiv b^ipmod{frac{b^p-1}{b-1}}$ for some $ileq p-1$. Finally, if $anot= b^i$, then $b^{p-1}-1geq |a-b^i|geqfrac{b^p-1}{b-1}$. Contradiction. $Box$
                – solver6
                Nov 27 '18 at 21:26


















              Can someone find any mistakes in another proof : Let $p$ be a prime number greater than $a$. Consider a polynomial $F(x)=x^p-1$ over $mod{frac{b^p-1}{b-1}}$. One can get that it has $d$ consecutive roots $1, b,ldots , b^{p-1}$. So from the fact $b^p-1mid a^p-1$ we get that $a$ is also root of $F(x)equiv 0pmod{frac{b^p-1}{b-1}}$. Thus, $aequiv b^ipmod{frac{b^p-1}{b-1}}$ for some $ileq p-1$.
              – solver6
              Nov 27 '18 at 21:19




              Can someone find any mistakes in another proof : Let $p$ be a prime number greater than $a$. Consider a polynomial $F(x)=x^p-1$ over $mod{frac{b^p-1}{b-1}}$. One can get that it has $d$ consecutive roots $1, b,ldots , b^{p-1}$. So from the fact $b^p-1mid a^p-1$ we get that $a$ is also root of $F(x)equiv 0pmod{frac{b^p-1}{b-1}}$. Thus, $aequiv b^ipmod{frac{b^p-1}{b-1}}$ for some $ileq p-1$.
              – solver6
              Nov 27 '18 at 21:19












              Can someone find any mistakes in another proof : Let $p$ be a prime number greater than $a$. Consider a polynomial $F(x)=x^p-1$ over $mod{frac{b^p-1}{b-1}}$. One can get that $P(x)$ has $d$ consecutive roots $1, b,ldots , b^{p-1}$. So from the fact $b^p-1mid a^p-1$ we get that $a$ is also root of $F(x)equiv 0pmod{frac{b^p-1}{b-1}}$. Thus, $aequiv b^ipmod{frac{b^p-1}{b-1}}$ for some $ileq p-1$. Finally, if $anot= b^i$, then $b^{p-1}-1geq |a-b^i|geqfrac{b^p-1}{b-1}$. Contradiction. $Box$
              – solver6
              Nov 27 '18 at 21:26






              Can someone find any mistakes in another proof : Let $p$ be a prime number greater than $a$. Consider a polynomial $F(x)=x^p-1$ over $mod{frac{b^p-1}{b-1}}$. One can get that $P(x)$ has $d$ consecutive roots $1, b,ldots , b^{p-1}$. So from the fact $b^p-1mid a^p-1$ we get that $a$ is also root of $F(x)equiv 0pmod{frac{b^p-1}{b-1}}$. Thus, $aequiv b^ipmod{frac{b^p-1}{b-1}}$ for some $ileq p-1$. Finally, if $anot= b^i$, then $b^{p-1}-1geq |a-b^i|geqfrac{b^p-1}{b-1}$. Contradiction. $Box$
              – solver6
              Nov 27 '18 at 21:26













              0














              Reference for a generalization of this theorem and it's proof in French. See this article.



              Langlois, Bruno, An arithmetical application of Hadamard’s quotient theorem, Ann. Univ. Sci. Budap. Rolando Eötvös, Sect. Comput. 44, 183-196 (2015). ZBL1389.11046.






              share|cite|improve this answer


























                0














                Reference for a generalization of this theorem and it's proof in French. See this article.



                Langlois, Bruno, An arithmetical application of Hadamard’s quotient theorem, Ann. Univ. Sci. Budap. Rolando Eötvös, Sect. Comput. 44, 183-196 (2015). ZBL1389.11046.






                share|cite|improve this answer
























                  0












                  0








                  0






                  Reference for a generalization of this theorem and it's proof in French. See this article.



                  Langlois, Bruno, An arithmetical application of Hadamard’s quotient theorem, Ann. Univ. Sci. Budap. Rolando Eötvös, Sect. Comput. 44, 183-196 (2015). ZBL1389.11046.






                  share|cite|improve this answer












                  Reference for a generalization of this theorem and it's proof in French. See this article.



                  Langlois, Bruno, An arithmetical application of Hadamard’s quotient theorem, Ann. Univ. Sci. Budap. Rolando Eötvös, Sect. Comput. 44, 183-196 (2015). ZBL1389.11046.







                  share|cite|improve this answer












                  share|cite|improve this answer



                  share|cite|improve this answer










                  answered Jan 3 at 21:03









                  solver6

                  5917




                  5917






























                      draft saved

                      draft discarded




















































                      Thanks for contributing an answer to Mathematics Stack Exchange!


                      • Please be sure to answer the question. Provide details and share your research!

                      But avoid



                      • Asking for help, clarification, or responding to other answers.

                      • Making statements based on opinion; back them up with references or personal experience.


                      Use MathJax to format equations. MathJax reference.


                      To learn more, see our tips on writing great answers.





                      Some of your past answers have not been well-received, and you're in danger of being blocked from answering.


                      Please pay close attention to the following guidance:


                      • Please be sure to answer the question. Provide details and share your research!

                      But avoid



                      • Asking for help, clarification, or responding to other answers.

                      • Making statements based on opinion; back them up with references or personal experience.


                      To learn more, see our tips on writing great answers.




                      draft saved


                      draft discarded














                      StackExchange.ready(
                      function () {
                      StackExchange.openid.initPostLogin('.new-post-login', 'https%3a%2f%2fmath.stackexchange.com%2fquestions%2f1964788%2fif-a-geq-2-a-nmid-b-and-an-1-mid-bn-1-for-all-n-in-mathbbn-then%23new-answer', 'question_page');
                      }
                      );

                      Post as a guest















                      Required, but never shown





















































                      Required, but never shown














                      Required, but never shown












                      Required, but never shown







                      Required, but never shown

































                      Required, but never shown














                      Required, but never shown












                      Required, but never shown







                      Required, but never shown







                      Popular posts from this blog

                      An IMO inspired problem

                      Management

                      Has there ever been an instance of an active nuclear power plant within or near a war zone?